Forme quadratique et dérivées partielles

Partie

Question

Soient \(E\) un espace vectoriel de dimension finie \(n\) sur le corps \(\mathbf R\), \(B=(e_1,e_2,...,e_n)\) une base de \(E\), \(q\) une forme quadratique sur \(E\) et \(f\) la forme bilinéaire symétrique associée à \(q\).

On considère l'application \(\Phi\) définie par :

\(\begin{array}{ccccl}\Phi&:&\mathbf R^n &\rightarrow& \mathbf R\\&&x=(x_1,x_2,...,x_n)&\mapsto&\displaystyle{\Phi(x)=q\left(\sum_{1\le i\le n}x_ie_i\right)}\end{array}\)

Montrer, pour tout \(x=(x_1,x_2,...,x_n)\in\mathbf R^n\), tout \(y=(y_1,y_2,...,y_n)\in\mathbf R^n\) et tout entier \(k\) compris entre \(1\) et \(n\), les égalités :

i. \(\displaystyle{\frac{1}{2}\frac{\partial\Phi}{\partial x_k}(x)=\sum_{1\le i\le n}f(e_i,e_k)x_i}\)

ii. \(\displaystyle{f\left(\sum_{1\le i\le n}x_ie_i,\sum_{1\le j\le n}y_je_j\right)=\frac{1}{2}\sum_{1\le k\le n}\left(y_k\frac{\partial\Phi}{\partial x_k}(x)\right)}\)

Applications :

a. Soit \(E=\mathbf R^3\). On considère la forme quadratique \(q\) définie pour tout \(x=(x_1,x_2,x_3)\) par \(q(x)={x_1}^2+2{x_2}^2+3x_1x_2+2x_2x_3\).

Déterminer la forme bilinéaire symétrique associée à la forme quadratique \(q\) et la matrice associée à \(q\) dans la base canonique.

b. Soient \(E\) un espace vectoriel de dimension finie \(n\) sur le corps \(\mathbf R\) et \(B=(e_1,e_2,...,e_n)\) une base de \(E\).

Vérifier que l'application \(Q\) de \(E\) dans \(\mathbf R\) définie par :

\(\displaystyle{Q\left(\sum_{1\le i\le n}x_ie_i\right)=\sum_{1\le i<j \le n}(x_i-x_j)^2}\)

est une forme quadratique sur \(E\) et déterminer sa matrice dans la base \(B\).

Aide simple

Développer \(\displaystyle{q\left(\sum_{1\le i \le n}x_ie_i\right)}\) isolant tous les termes en \(x_k\), pour déterminer la dérivée partielle de \(\Phi\) par rapport \(x_k\).

Aide à la lecture

L'application \(q\) est une forme quadratique définie sur un espace vectoriel \(E\) de dimension \(n\) sur \(\mathbf K\) tandis que l'application \(\Phi\) est définie sur \(\mathbf K^n\); on considère alors les dérivées partielles de \(\Phi\) par rapport aux \(x_k\).

Le but de l'exercice est de trouver une relation entre ces dérivées partielles et les coefficients de la matrice de \(q\).

Solution détaillée
  1. L'image par \(q\) du vecteur \(\displaystyle{\sum_{1\le i\le n}x_ie_i}\) est : \(\displaystyle{q\left(\sum_{1\le i\le n}x_ie_i\right)=\sum_{1\le i\le n}f(e_i,e_i){x_i}^2+2\sum_{1\le i<j \le n}f(e_i,e_j)x_ix_j}\).

    Donc pour tout \(x=(x_1,x_2,...,x_n)\in\mathbf K^n\), \(\displaystyle{\Phi(x)=\sum_{1\le i\le n}f(e_i,e_i){x_i}^2+2\sum_{1\le i<j\le n}f(e_i,e_j)x_ix_j}.(*)\).

    Soit \(k\) un entier fixé compris entre \(1\) et \(n\). Pour calculer \(\frac{\partial \Phi}{\partial x_k}\), il faut repérer dans l'expression \((*)\) les termes où apparaît \(x_k\).

    En isolant le scalaire \(x_k\) on a : \(\Phi(x)=\Phi_1(x)+\Phi_2(x)\), où :

    \(\left\{\begin{array}{rcl}\Phi_1(x)&=&\displaystyle{f(e_k,e_k){x_k}^2+2\sum_{1\le i< k}f(e_i,e_k)x_ix_k+2\sum_{k<i\le n}f(e_i,e_k)x_ix_k}\\\\\Phi_2(x)&=&\displaystyle{\sum_{\substack{{1\le i\le n}\\{i\neq k}}}f(e_i,e_i){x_i}^2+2\sum_{\substack{{1\le i<j\le n}\\{i\neq k,~~j\neq k}}}f(e_i,e_j)x_ix_j}\end{array}\right.\)

    Dans l'expression \(\Phi_2(x)\), le scalaire \(x_k\) n'intervient pas.

    Par conséquent \(\frac{\partial\Phi}{\partial x_k}(x)=\frac{\partial\Phi_1}{\partial x_k}(x)\), ce qui donne :

    \(\frac{\partial\Phi}{\partial x_k}(x)=\displaystyle{2f(e_k,e_k){x_k}+2\sum_{1\le i< k}f(e_i,e_k)x_i+2\sum_{k<i\le n}f(e_i,e_k)x_i}\).

    D'où :

    \(\frac{\partial\Phi}{\partial x_k}(x)=\displaystyle{2\sum_{1\le i\le n}f(e_i,e_k)x_i}\).

    On a donc démontré que pour tout \(x=(x_1,x_2,...,x_n)\in\mathbf K^n\)et tout entier \(k\), \(1\le k\le n\):

    \(\frac{1}{2}\frac{\partial\Phi}{\partial x_k}(x)=\displaystyle{\sum_{1\le i\le n}f(e_i,e_k)x_i}\).

    On a aussi :

    \(\begin{array}{rcl}\displaystyle{\frac{1}{2}\sum_{1\le k\le n}\left(y_k\frac{\partial\Phi}{\partial x_k}(x)\right)}&=&\displaystyle{\sum_{1\le k\le n}\left(y_k\frac{1}{2}\frac{\partial\Phi}{\partial x_k}(x)\right)}\\&=&\displaystyle{\sum_{1\le k\le n}\left(y_k\sum_{1\le i\le n}f(e_i,e_k)x_i\right)}\end{array}\)

    d'après le résultat précédent.

    Donc, en utilisant la bilinéarité de \(f\), on obtient :

    \(\begin{array}{rcl}\displaystyle{\frac{1}{2}\sum_{1\le k\le n}\left(y_k\frac{\partial\Phi}{\partial x_k}(x)\right)}&=&\displaystyle{\sum_{1\le k\le n}\left(y_k\sum_{1\le i\le n}f(e_i,e_k)x_i\right)}\\\\&=&\displaystyle{\sum_{1\le k\le n}\left(\sum_{1\le i\le n}f(x_ie_i,y_ke_k)\right)}\\\\&=&\displaystyle{\sum_{1\le k\le n}f\left(\sum_{1\le i\le n}x_ie_i,y_ke_k\right)}\\\\&=&\displaystyle{f\left(\sum_{1\le k\le n}x_ie_i,\sum_{1\le i\le n}y_ke_k\right)}\end{array}\)

    On a donc démontré :

    \(\displaystyle{\frac{1}{2}\sum_{1\le k\le n}\left(y_k\frac{\partial\Phi}{\partial x_k}(x)\right)}=\displaystyle{f\left(\sum_{1\le k\le n}x_ie_i,\sum_{1\le i\le n}y_ke_k\right)}\)

  2. Applications

    • a) Pour \(x=(x_1,x_2,x_3)\) dans \(\mathbf R^3\), \(q(x)={x_1}^2+2{x_2}^2+3x_1x_2+2x_2x_3\).

      Dans cet exemple, \(q\) et \(\Phi\) sont identiques.

      En notant \((e_1,e_2,e_3)\) la base canonique de \(\mathbf R^3\), d'après le résultat \(\displaystyle{f\left(\sum_{1\le k\le n}x_ie_i,\sum_{1\le i\le n}y_ke_k\right)}=\displaystyle{\frac{1}{2}\sum_{1\le k\le n}\left(y_k\frac{\partial\Phi}{\partial x_k}(x)\right)}\), on a pour \(x=(x_1,x_2,x_3)\) et \(y=(y_1,y_2,y_3)\) dans \(\mathbf R^3\):

      \(f(x,y)=\frac{1}{2}\left(y_1\frac{\partial q}{\partial x_1}(x)+y_2\frac{\partial q}{\partial x_2}(x)+y_3\frac{\partial q}{\partial x_3}(x)\right)\).

      Les trois dérivées partielles de \(q\) sont : \(\frac{\partial q}{\partial x_1}(x)=2x_1+3x_2\), \(\frac{\partial q}{\partial x_2}(x)=3x_1+4x_2+2x_3\), \(\frac{\partial q}{\partial x_3}(x)=2x_2\), donc \(f(x,y)=\frac{1}{2}\left(2x_1y_1+3x_2y_1+3x_1y_2+4x_2y_2+2x_3y_2+2x_2y_3\right)\), par conséquent :

      \(f(x,y)=x_1y_1+\frac{3}{2}(x_2y_1+x_1y_2)+2x_2y_2+x_3y_2+x_2y_3\).

      La matrice associée à la forme quadratique q dans la base canonique est donc

      \(\left(\begin{array}{ccc}1&3/2&0\\3/2&2&1\\0&1&0\end{array}\right)\).

    • b) Soit \(Q\) l'application de \(E\) dans \(\mathbf K\) définie par \(\displaystyle{Q\left(\sum_{1\le i\le n}x_ie_i\right)=\sum_{1\le i<j\le n}(x_i-x_j)^2}\).

      En développant le deuxième membre de cette égalité, on obtient un polynôme homogène de degré 2 par rapport aux coordonnées \(x_i\), donc \(Q\) est une forme quadratique.

      Soit \(\Psi\) l'application définie par :

      \(\begin{array}{ccccl}\Psi &:& \mathbf K^n &\rightarrow& \mathbf K\\&&x=(x_1,x_2,...,x_n) &\mapsto& \Psi(x)=\sum_{1\le i<j \le n}(x_i-x_j)^2\end{array}\)

      D'après la question précédente, en notant \(g\) la forme bilinéaire symétrique associée à \(Q\), on a pour tout \(x=(x_1,x_2,...,x_n)\in\mathbf K^n\) et tout entier \(k\), \(1\le k \le n\):

      \(\displaystyle{\frac{1}{2}\frac{\partial\Psi}{\partial x_k}(x)=\sum_{1\le i\le n}g(e_i,e_k)x_i}\).

      Soit \(k\) un entier fixé compris entre \(1\) et \(n\). Pour déterminer \(\frac{\partial\Psi}{\partial x_k}\), on écrit \(\Psi(x)\) sous la forme suivante :

      \(\displaystyle{\Psi(x)=\sum_{1\le i<k}(x_i-x_k)^2+\sum_{k<j\le n}(x_k-x_j)^2+\sum_{\substack{{1\le i<j \le n}\\{i\neq k,~~j\neq k}}}(x_i-x_j)^2}\)

      D'où

      \(\begin{array}{rcrl}\frac{\partial\Psi}{\partial x_k}(x)&=&-&\displaystyle{2\sum_{1\le i<k}(x_i-x_k)+2\sum_{k<j\le n}(x_k-x_j)}\\\\&=&&\displaystyle{2\sum_{\substack{{1\le i\le n}\\{i\neq k}}}(x_i-x_k)}\\\\&=&&\displaystyle{2(n-1)x_k-2\sum_{\substack{{1\le i\le n}\\{i\neq k}}}x_i}\end{array}\)

      Donc pour tout \(x=(x_1,x_2,...,x_n)\in\mathbf K^n\), on a \(\displaystyle{\frac{1}{2}\frac{\partial\Psi}{\partial x_k}(x)=\sum_{1\le i\le n}g(e_i,e_k)x_i=(n-1)x_k-\sum_{\substack{{1\le i\le n}\\{i\neq k}}}x_i}\)

      d'où \(g(e_i,e_k)=\left\{\begin{array}{ccl}n-1 &si& i=k\\-1 &si& i\neq k\end{array}\right.\)

      La matrice de \(Q\) dans la base \(B\) est donc :

      \(M(Q,B)=\left(\begin{array}{cccc}n-1&-1&\cdots&-1\\-1&n-1&\cdots&-1\\\vdots&\vdots&\ddots&\vdots\\-1&-1&\cdots&n-1\end{array}\right)\)